Diễn Đàn MathScopeDiễn Đàn MathScope
  Diễn Đàn MathScope
Ghi Danh Hỏi/Ðáp Community Lịch

Go Back   Diễn Đàn MathScope > Sơ Cấp > Việt Nam và IMO > 2013

News & Announcements

Ngoài một số quy định đã được nêu trong phần Quy định của Ghi Danh , mọi người tranh thủ bỏ ra 5 phút để đọc thêm một số Quy định sau để khỏi bị treo nick ở MathScope nhé !

* Nội quy MathScope.Org

* Một số quy định chung !

* Quy định về việc viết bài trong diễn đàn MathScope

* Nếu bạn muốn gia nhập đội ngũ BQT thì vui lòng tham gia tại đây

* Những câu hỏi thường gặp

* Về việc viết bài trong Box Đại học và Sau đại học


Trả lời Gởi Ðề Tài Mới
 
Ðiều Chỉnh Xếp Bài
Old 01-02-2013, 12:00 AM   #1
namdung
Administrator

 
Tham gia ngày: Feb 2009
Đến từ: Tp Hồ Chí Minh
Bài gởi: 1,343
Thanks: 209
Thanked 4,066 Times in 778 Posts
Gửi tin nhắn qua Yahoo chát tới namdung
Tiến tới kỳ thi Vietnam TST 2013

Chào các bạn,

Như vậy danh sách các thí sinh tham dự kỳ thi chọn đội tuyển toán dự IMO 2013 đã được xác định. Họ sẽ tiếp tục tranh tài với nhau để chọn ra 6 bạn có phong độ tốt nhất đại diện cho Việt Nam tại kỳ thi toán lớn nhất dành cho học sinh toàn thế giới.

Để chuẩn bị cho kỳ thi này, giúp các thí sinh có điều kiện làm quen với các bài toán "mức độ TST", cũng như thảo luận một số bài toán của các kỳ TST trước đó, bổ sung một số kiến thức và kỹ năng cần thiết cho các thí sinh, chúng tôi lập ra chủ đề này.

Chúng tôi sẽ lần lượt gửi lên các đề toán với nhiều mức độ: trung bình, tương đối khó, khó và rất khó thuộc về 4 phân môn: Đại số, Hình học, Số học và Tổ hợp. Chúng ta sẽ cùng giải, thảo luận, bình luận về cách giải, về phương pháp giải, về các kiến thức lý thuyết liên quan, và nêu các bài toán cùng dạng.

Chú ý rằng kinh nghiệm của các kỳ TST trước cho thấy, để lọt vào đội tuyển thì phải làm được các bài trung bình và tương đối khó một cách hoàn hảo và kiếm điểm được ở những bài khó, trong đó vế đầu gần như là điều kiện tiên quyết. Vì thế các bạn cũng chú ý giải cẩn thận và chi tiết những bài thuộc loại trung bình và cũng đừng quá hoang mang khi thấy một số bài gần như "ngoài vùng phủ sóng" của các bạn.

Chúng ta bắt đầu từ 4 bài toán đầu tiên (làm trong 450 phút):

1. Cho 0 < a, b, c, d < 1 và abcd = (1-a)(1-b)(1-c)(1-d). Chứng minh rằng
$ (a+b+c+d) - (a+c)(b+d) \ge 1 $

2. Chứng minh rằng với mọi số nguyên dương n > 1 tổng $1^{1} + {3}^{3} + {5}^{5} + ... + {(2^n-1)^{2^n-1} $ chia hết cho $2^n $ nhưng không chia hết cho $2^{n+1} $

3. Cho tam giác nhọn ABC. M, N là trung điểm các cạnh AB, AC tương ứng và gọn P là hình chiếu của N lên cạnh BC và $A_1 $ là trung điểm của MP. Các điểm $B_1, C_1 $ được xây dựng một cách tương tự. Chứng minh rằng nếu $AA_1, BB_1, CC_1 $ đồng quy tại một điểm thì tam giác ABC cân.

4. Một đường tròn màu xanh được chia bởi 100 điểm màu đỏ thành các cung có độ dài theo một thứ tự tùy ý nào đó là 1, 2, ..., 100. Chứng minh rằng tồn tại một đường kính có hai đầu mút màu đỏ.

Chủ đề mong nhận được sự ủng hộ và chung tay của các thầy giáo, các cựu IMO, cựu TST, cựu VMO để cùng hướng dẫn, chia sẻ kinh nghiệm.

Chú ý khi post đề bài lên cần cân nhắc và chọn lọc kỹ càng, với mỗi bài toán nên gắn liền với một phương pháp, một kỹ thuật cụ thể mà các bạn muốn truyền đến cho các bạn thí sinh. Gọi là Quý hồ tinh bất quý hồ đa.
[RIGHT][I][B]Nguồn: MathScope.ORG[/B][/I][/RIGHT]
 
namdung is offline   Trả Lời Với Trích Dẫn
The Following 33 Users Say Thank You to namdung For This Useful Post:
anhdunghmd (01-02-2013), blackholes. (01-02-2013), boykhtna1 (19-02-2013), Conan Edogawa (25-02-2013), Conanvn (03-02-2013), CTK9 (28-01-2014), DaiToan (01-02-2013), Dongcdhv (01-02-2013), dvtruc (04-02-2013), Gin Mellkior (01-02-2013), hoanghaithanh (26-02-2013), huynhcongbang (01-02-2013), kieudinhminh (02-02-2013), luugiangnam (07-03-2013), Mai Nguyen (05-02-2013), Mr_Pi (03-02-2013), nghiepdu-socap (01-02-2013), nguoilamat01 (01-02-2013), nguyentatthu (01-02-2013), nyctkt (01-02-2013), pHnAM (01-02-2013), pmn_t1114 (01-02-2013), sang_zz (20-02-2013), thaygiaocht (01-02-2013), thiendienduong (01-02-2013), thinhso01 (28-02-2013), TNP (01-02-2013), transonlvt (18-02-2013), Trànvănđức (01-02-2013), triethuynhmath (03-02-2013), tsunajudaime (01-02-2013), vinhhop.qt (01-02-2013), zớt (26-02-2013)
Old 01-02-2013, 06:36 AM   #2
huynhcongbang
Administrator

 
huynhcongbang's Avatar
 
Tham gia ngày: Feb 2009
Đến từ: Ho Chi Minh City
Bài gởi: 2,413
Thanks: 2,165
Thanked 4,188 Times in 1,381 Posts
Gửi tin nhắn qua Yahoo chát tới huynhcongbang
Em xin giải bài 3.

Bài này cũng xuất hiện trong khá nhiều tài liệu, thuộc về nội dung hình học tính toán. Lúc trước em có phát biểu lại một tí nhưng vấn đề cơ bản thì vẫn thế. Dưới đây là lời giải chi tiết.

Cho tam giác $ABC$ nhọn, không cân có diện tích S và có $M, N$ lần lượt là trung điểm $AB, AC.$ Với $x\in \left( 0;1 \right)$, xét điểm $M, N$ lần lượt thuộc các cạnh $AB, AC$ sao cho $\frac{AM}{AB}=\frac{AN}{AC}=x$ và gọi $X$ tâm của hình chữ nhật ${{R}_{1}}$ nội tiếp tam giác $ABC$ với $M, N$ là hai đỉnh của hình chữ nhật ${{R}_{1}}$ và hai đỉnh còn lại thuộc cạnh $BC$. Ứng với hai số thực $y,z\in \left( 0;1 \right)$, ta lần lượt xét các hình chữ nhật ${{R}_{2}},{{R}_{3}}$ với tâm của chúng theo thứ tự là $Y$ và $Z$ với cách xác định tương tự.
a) Chứng minh rằng ${{S}_{{{R}_{1}}}}{{S}_{{{R}_{2}}}}{{S}_{{{R}_{3}} }}\le \frac{1}{8}S^3$.
b) Trong trường hợp dấu đẳng thức xảy ra trong bất đẳng thức trên, chứng minh rằng các đường thẳng $AX, BY, CZ$ không đồng quy.

Lời giải.

a) Giả sử hình chữ nhật tương ứng có tâm $X$ là $MNPQ$ với $P, Q$ thuộc cạnh $BC$.
Theo định lí Thales thì:
$\frac{MN}{BC}=\frac{AM}{AB}=x$ và $\frac{MQ}{AH}=\frac{BM}{AB}=1-\frac{AM}{AB}=1-x$
trong đó $H$ là hình chiếu của $A$ lên cạnh $BC$.
Suy ra ${{S}_{MNPQ}}=MN.MQ=x(1-x).AH.BC=2x(1-x)S$.
Theo bất đẳng thức Cauchy cho hai số không âm $x,1-x$ thì $x(1-x)\le \frac{{{(x+1-x)}^{2}}}{4}=\frac{1}{4}$ nên ta có ${{S}_{{{R}_{1}}}}=2x(1-x)S\le \frac{1}{2}S$.
Tương tự, ta cũng có ${{S}_{{{R}_{2}}}}\le \frac{1}{2}S,{{S}_{{{R}_{3}}}}\le \frac{1}{2}S$. Nhân các bất đẳng thức này lại, ta được đpcm.


b) Dễ thấy rằng dấu bằng xảy ra trong các bất đẳng thức trên khi và chỉ khi $x=y=z=\frac{1}{2}$ hay cứ mỗi hình chữ nhật thì có một cạnh là đường trung bình của tam giác ABC.
Ta sẽ tính tỉ lệ $\dfrac{\sin \angle XAB}{\sin \angle XAC}$ theo độ dài các cạnh của tam giác $ABC$. Đặt $BC=a,CA=b,AB=c.$
Ta có $$\frac{{{S}_{AMX}}}{{{S}_{ANX}}}=\frac{AM.AX.\sin \angle XAB}{AN.AX.\sin \angle XAC}\Rightarrow \frac{\sin \angle XAB}{\sin \angle XAB}=\frac{{{S}_{AMX}}}{{{S}_{ANX}}}\frac{b}{c}. $$ Hơn nữa, theo tỉ lệ diện tích các tam giác, ta có $${{S}_{AMX}}=\frac{1}{2}{{S}_{AMP}}=\frac{1}{4}{{ S}_{ABP}}=\frac{BP}{BC}S=\frac{a-\frac{b}{2}\cos C}{4a}S=\frac{a-\frac{b}{2}\frac{{{a}^{2}}+{{b}^{2}}-{{c}^{2}}}{2ab}}{4a}S=\frac{3{{a}^{2}}-{{b}^{2}}+{{c}^{2}}}{16{{a}^{2}}}S. $$
Tương tự, ta cũng tính được
${{S}_{ANX}}=\dfrac{3{{a}^{2}}+{{b}^{2}}-{{c}^{2}}}{16{{a}^{2}}}S$ nên $\dfrac{\sin \angle XAB}{\sin \angle XAC}=\dfrac{3{{a}^{2}}-{{b}^{2}}+{{c}^{2}}}{3{{a}^{2}}+{{b}^{2}}-{{c}^{2}}}\cdot \frac{b}{c}$.
Xây dựng các đẳng thức tương ứng về góc đối với Y và Z rồi áp dụng định lí Ceva, ta thấy điều kiện để các đoạn $AX, BY, CZ$ đồng quy khi và chỉ khi $$\prod\limits_{sym}{\left( \frac{3{{a}^{2}}-{{b}^{2}}+{{c}^{2}}}{3{{a}^{2}}+{{b}^{2}}-{{c}^{2}}}\cdot \frac{b}{c} \right)}=1\Rightarrow \prod\limits_{sym}{\left( \frac{3{{a}^{2}}-{{b}^{2}}+{{c}^{2}}}{3{{a}^{2}}+{{b}^{2}}-{{c}^{2}}} \right)}=1\Rightarrow \prod\limits_{sym}{\left( 3{{a}^{2}}-{{b}^{2}}+{{c}^{2}} \right)}=\prod\limits_{sym}{\left( 3{{a}^{2}}+{{b}^{2}}-{{c}^{2}} \right)}$$ Giả sử $P(a,b,c)=\prod\limits_{sym}{\left( 3{{a}^{2}}-{{b}^{2}}+{{c}^{2}} \right)}-\prod\limits_{sym}{\left( 3{{a}^{2}}+{{b}^{2}}-{{c}^{2}} \right)}$, ta thấy rằng đây là đa thức có bậc không quá 6 và hơn nữa $P(a,t,t)=P(a,t,-t)=P(t,b,t)=P(t,b,-t)=P(t,t,c)=P(t,-t,c)=0$ nên nó có thể phân tích thành nhân tử dạng $P(a,b,c)=\gamma (a-b)(b-c)(c-a)(a+b)(b+c)(c+a)$ với $\gamma $ là một số thực nào đó.

Vì tam giác $ABC$ không cân nên $(a-b)(b-c)(c-a)\ne 0$ và do đó, đẳng thức trên không thể xảy ra được. Vậy các đoạn thẳng $AX, BY, CZ$ không đồng quy.
Ta có đpcm.

Em ủng hộ tiếp 4 bài sau:

Bài 5.

Cho dãy số $({{u}_{n}})$ xác định bởi $\left\{ \begin{align}
& {{a}_{0}}={{a}_{1}}=1, \\
& {{a}_{n+1}}=\frac{(2n+3){{a}_{n}}+3n{{a}_{n-1}}}{n+3},n\ge 1. \\
\end{align} \right.$
Chứng minh rằng tất cả số hạng của dãy đều nguyên.

Bài 6.

Với mỗi số nguyên dương $n,$ gọi ${{a}_{n}}$ là tổng số lượng các chữ số của ${{2}^{n}}$ và ${{5}^{n}}$ khi viết trong hệ thập phân. Hãy tìm tất cả các số nguyên dương $m$ sao cho tồn tại một hoán vị của các số ${{a}_{1}},{{a}_{2}},{{a}_{3}},...,{{a}_{m}}$ mà tổng của $k$ số hạng đầu tiên trong dãy hoán vị đó không chia hết cho $k$ với mọi $k=2,3,...,m$.

Bài 7.

Cho các số nguyên tố ${{p}_{1}},{{p}_{2}},{{p}_{3}},...,{{p}_{k}}$ đôi một khác nhau và cho các số tự nhiên bất kì ${{n}_{1}},{{n}_{2}},{{n}_{3}},...,{{n}_{k}}$ đều lớn hơn 2. Chứng minh rằng số các cặp $(x,y)$ không có thứ tự, nguyên tố cùng nhau và thỏa mãn đẳng thức sau $${{x}^{7}}+{{y}^{7}}=p_{1}^{{{n}_{1}}}\cdot p_{2}^{{{n}_{2}}}\cdot p_{3}^{{{n}_{3}}}\cdot ...\cdot p_{k}^{{{n}_{k}}}$$ không vượt quá ${{2}^{k+1}}.$


Bài 8.

Xét số nguyên dương $n>2.$ Cho $n$ túi kẹo, mỗi túi có đúng 1 viên kẹo và hai người $A,B$ chơi một trò chơi như sau: Ở mỗi lượt chơi, người chơi chọn hai túi kẹo tùy ý có số kẹo nguyên tố cùng nhau rồi gộp chúng lại thành một túi kẹo. Người nào không thực hiện được thao tác này thì thua cuộc.
Biết rằng $A$ là người đi trước và hai người chơi lần lượt, hỏi ai là người có chiến lược để thắng trò chơi này?
[RIGHT][I][B]Nguồn: MathScope.ORG[/B][/I][/RIGHT]
 
__________________
Sự im lặng của bầy mèo

thay đổi nội dung bởi: huynhcongbang, 01-02-2013 lúc 06:42 AM
huynhcongbang is offline   Trả Lời Với Trích Dẫn
The Following 15 Users Say Thank You to huynhcongbang For This Useful Post:
anhdunghmd (02-02-2013), CTK9 (28-01-2014), Dongcdhv (03-02-2013), dvtruc (04-02-2013), Gin Mellkior (01-02-2013), High high (01-02-2013), hoang_kkk (01-02-2013), lexuanthang (01-02-2013), Mr_Pi (03-02-2013), namdung (01-02-2013), nghiepdu-socap (01-02-2013), nyctkt (01-02-2013), Trànvănđức (01-02-2013), triethuynhmath (03-02-2013), Yucio.3bi_love (18-02-2013)
Old 01-02-2013, 09:57 AM   #3
ThangToan
+Thành Viên+
 
Tham gia ngày: Nov 2010
Đến từ: THPT chuyên Vĩnh Phúc
Bài gởi: 570
Thanks: 24
Thanked 537 Times in 263 Posts
1. Cho 0 < a, b, c, d < 1 và abcd = (1-a)(1-b)(1-c)(1-d). Chứng minh rằng
$ (a+b+c+d) - (a+c)(b+d) \ge 1 $

Lời giải. Dùng phương pháp hình học.Dựng hình vuông $ABCD $ có cạnh bằng $1 $. Gọi $M, N, P, Q $ lần lượt là các điểm nằm trên các cạnh (không trùng với đỉnh) $AB, BC, CD, DA $ sao cho $AM=a, BN=b, CP=c, DQ=d $. Khi đó ta có:
$S(ABCD)=S(MNPQ)+S(AMQ)+S(BMN)+S(CNP)+S(DPQ) $
$ \Leftrightarrow 2S\left( {ABCD} \right) = 2S\left( {MNPQ} \right) + a\left( {1 - d} \right) + b\left( {1 - a} \right) + c\left( {1 - b} \right) + d\left( {1 - c} \right) $
$ \Leftrightarrow 2 = 2S\left( {MNPQ} \right) + a + b + c + d - \left( {a + c} \right)\left( {b + d} \right) $ (1).
Bây giờ ta sẽ chứng minh $2S(MNPQ)\le 1 $ (2). Chú ý cặp số $(a, c); (b,d) $ có vai trò như nhau.
TH1. Nếu $b\le \frac{1}{2}, d\ge \frac{1}{2} $ hoặc $d\le \frac{1}{2}, b\ge \frac{1}{2} $
Thật vậy, từ $N $ kẻ đường thẳng song song với $AB $ cắt $MP $ tại $N' $ và từ $Q $ kẻ đường thẳng song song với $AB $ cắt $MP $ tại $Q' $. Khi đó
$\[S\left( {MNPQ} \right) = S\left( {NMP} \right) + S\left( {QMP} \right) = \frac{1}{2}BC.NN' + \frac{1}{2}AD.QQ'\] $
$\[S\left( {MNPQ} \right) = \frac{1}{2}\left( {NN' + QQ'} \right) \le \frac{1}{2}AB = \frac{1}{2}\] $.
Do đó $(2) $ đúng suy ra $(1) $ đúng. Vậy bđt được chứng minh.
TH2. $min\{a, b, c, d\}\ge \frac{1}{2} $ thì từ giả thiết ta có ngay $a=b=c=d=\frac{1}{2} $
Th3. $min\{b, d\}\ge \frac{1}{2}, max\{a, c\}\le \frac{1}{2} $
$\[a + b + c + d - \left( {a + c} \right)\left( {b + d} \right) \ge 1 \Leftrightarrow \left( {a + c - 1} \right)\left( {b + d - 1} \right) \le 0\] $ luôn đúng.
[RIGHT][I][B]Nguồn: MathScope.ORG[/B][/I][/RIGHT]
 

thay đổi nội dung bởi: ThangToan, 01-02-2013 lúc 10:27 AM
ThangToan is offline   Trả Lời Với Trích Dẫn
The Following 14 Users Say Thank You to ThangToan For This Useful Post:
CTK9 (28-01-2014), Gin Mellkior (01-02-2013), huynhcongbang (01-02-2013), lexuanthang (01-02-2013), Mr_Pi (03-02-2013), namdung (01-02-2013), nghiepdu-socap (01-02-2013), nguyentatthu (01-02-2013), nqt (02-02-2013), ntuan5 (01-02-2013), sang_zz (20-02-2013), Trànvănđức (01-02-2013), triethuynhmath (03-02-2013), vinhhop.qt (01-02-2013)
Old 01-02-2013, 10:08 AM   #4
namdung
Administrator

 
Tham gia ngày: Feb 2009
Đến từ: Tp Hồ Chí Minh
Bài gởi: 1,343
Thanks: 209
Thanked 4,066 Times in 778 Posts
Gửi tin nhắn qua Yahoo chát tới namdung
Trích:
Nguyên văn bởi huynhcongbang View Post
Bài 6.

Với mỗi số nguyên dương $n,$ gọi ${{a}_{n}}$ là tổng số lượng các chữ số của ${{2}^{n}}$ và ${{5}^{n}}$ khi viết trong hệ thập phân. Hãy tìm tất cả các số nguyên dương $m$ sao cho tồn tại một hoán vị của các số ${{a}_{1}},{{a}_{2}},{{a}_{3}},...,{{a}_{m}}$ mà tổng của $k$ số hạng đầu tiên trong dãy hoán vị đó không chia hết cho $k$ với mọi $k=2,3,...,m$.
Thầy không ủng hộ lắm cách ghép cơ học của hai bài toán không cùng bản chất như bài toán đề xuất của Lữ.

Thầy nghĩ bài này nên phát biểu là: Tìm tất cả các số nguyên dương n sao cho tồn tại một hoán vị của các số {2, 3, ..., n+1} sao cho tổng của k số hạng đầu tiên trong dãy đó không chia hết cho k với mọi k = 2, 3, ..., n.
[RIGHT][I][B]Nguồn: MathScope.ORG[/B][/I][/RIGHT]
 
namdung is offline   Trả Lời Với Trích Dẫn
The Following 7 Users Say Thank You to namdung For This Useful Post:
huynhcongbang (01-02-2013), mnnn (01-02-2013), Mr_Pi (03-02-2013), nguyentatthu (01-02-2013), thaygiaocht (01-02-2013), Trànvănđức (01-02-2013), Yucio.3bi_love (18-02-2013)
Old 01-02-2013, 10:35 AM   #5
12121993
+Thành Viên+
 
Tham gia ngày: Feb 2012
Bài gởi: 81
Thanks: 23
Thanked 70 Times in 41 Posts
1. Cho 0 < a, b, c, d < 1 và abcd = (1-a)(1-b)(1-c)(1-d). Chứng minh rằng
$ (a+b+c+d) - (a+c)(b+d) \ge 1 $

Giải:
đpcm tương đương với $(a+c - 1)(b+d - 1) \le 0 $
Từ đẳng thức $abcd = (1-a)(1-b)(1-c)(1-d) $ không mất tính tổng quát, giả sử
$ac \le (1-a)(1-c) $ và $bd \ge (1-b)(1-d) $
Khai triển và thu gọn ta được $a+c - 1 \le 0 $ và $b+d - 1 \ge 0 $
Từ đó ta có bdt cần chứng minh. Đẳng thức xảy ra khi và chỉ khi $a+c = b+d =1 $.
[RIGHT][I][B]Nguồn: MathScope.ORG[/B][/I][/RIGHT]
 
12121993 is offline   Trả Lời Với Trích Dẫn
The Following 5 Users Say Thank You to 12121993 For This Useful Post:
Mr_Pi (03-02-2013), namdung (01-02-2013), nguyentatthu (01-02-2013), nqt (02-02-2013), Yucio.3bi_love (18-02-2013)
Old 01-02-2013, 11:27 AM   #6
kien10a1
+Thành Viên+
 
kien10a1's Avatar
 
Tham gia ngày: Feb 2011
Đến từ: Vĩnh Yên- Vĩnh Phúc
Bài gởi: 371
Thanks: 43
Thanked 263 Times in 153 Posts
Gửi tin nhắn qua Yahoo chát tới kien10a1
Em góp một bài nhỏ:
Bài 9: Cho tam giác $ABC $ nhọn nội tiếp đường tròn $(O) $ với các đường cao $BE,CF $ và trực tâm $H $. $AO $ cắt $BC $ tại $K $, đường thẳng chứa trung tuyến của tam giác $HEF $ tại $H $ cắt $BC $ tại $L $. Chứng minh rằng: $K,L,E,F $ cùng thuộc một đường tròn.
[RIGHT][I][B]Nguồn: MathScope.ORG[/B][/I][/RIGHT]
 
__________________
Quay về với nơi bắt đầu

thay đổi nội dung bởi: kien10a1, 01-02-2013 lúc 08:35 PM
kien10a1 is offline   Trả Lời Với Trích Dẫn
The Following User Says Thank You to kien10a1 For This Useful Post:
Trànvănđức (01-02-2013)
Old 01-02-2013, 11:32 AM   #7
huynhcongbang
Administrator

 
huynhcongbang's Avatar
 
Tham gia ngày: Feb 2009
Đến từ: Ho Chi Minh City
Bài gởi: 2,413
Thanks: 2,165
Thanked 4,188 Times in 1,381 Posts
Gửi tin nhắn qua Yahoo chát tới huynhcongbang
Trích:
Nguyên văn bởi namdung View Post
Thầy không ủng hộ lắm cách ghép cơ học của hai bài toán không cùng bản chất như bài toán đề xuất của Lữ.

Thầy nghĩ bài này nên phát biểu là: Tìm tất cả các số nguyên dương n sao cho tồn tại một hoán vị của các số {2, 3, ..., n+1} sao cho tổng của k số hạng đầu tiên trong dãy đó không chia hết cho k với mọi k = 2, 3, ..., n.
Dạ, bài này em lấy nguyên gốc bài 1.2 trong cuốn sách của anh Lê Quang Nẫm đó thầy. Đúng là nếu để dưới dạng các số $2, 3, 4, ..., n+1$ thì sẽ thuần túy hơn nhưng cái ý: "số chữ số của $2^n$ và $5^n$ viết trong hệ thập phân là $n+1$" thì không phải ai cũng biết nên có gì giới thiệu cho mọi người cũng tốt ạ.
[RIGHT][I][B]Nguồn: MathScope.ORG[/B][/I][/RIGHT]
 
__________________
Sự im lặng của bầy mèo
huynhcongbang is offline   Trả Lời Với Trích Dẫn
Old 01-02-2013, 12:08 PM   #8
NhamNgaHanh
Vọng Phong Nhi Đào
 
NhamNgaHanh's Avatar
 
Tham gia ngày: Jul 2011
Bài gởi: 282
Thanks: 85
Thanked 207 Times in 111 Posts
Anh ấy nói đúng rồi đấy Lữ ạ!
[RIGHT][I][B]Nguồn: MathScope.ORG[/B][/I][/RIGHT]
 
__________________
Nhâm Ngã Hành
NhamNgaHanh is offline   Trả Lời Với Trích Dẫn
The Following User Says Thank You to NhamNgaHanh For This Useful Post:
huynhcongbang (21-02-2013)
Old 01-02-2013, 04:12 PM   #9
Traum
Moderator
 
Traum's Avatar
 
Tham gia ngày: Nov 2007
Đến từ: cyber world
Bài gởi: 413
Thanks: 14
Thanked 466 Times in 171 Posts
số số chính phương từ hoán vị

Bài 10: Cho $a_1,a_2,...a_{63} $ là một hoán vị của $1,2,...,63 $. Chứng minh rằng có không quá 38 số chính phương xuất hiện trong dãy: $b_k = 2017 - a_1-\cdots -a_k $, $k = 1,2,...,63 $. (Độ khó = nằm giữa bài 1 và bài 2 theo kiểu IMO hay TST)

Bài 11: Tìm tất cả hàm số $f:Q^+\to Q^+ $ thỏa mãn: $f(2) = 2 $, $f(xy) = f(x)f(y) $ và $f(x+y)\le 4f(x) + 2013f(y) $ với mọi $x,y\in Q^+ $. (Độ khó = bài 2 IMO hay TST).
[RIGHT][I][B]Nguồn: MathScope.ORG[/B][/I][/RIGHT]
 
__________________
Traum is giấc mơ.

thay đổi nội dung bởi: Traum, 01-02-2013 lúc 05:31 PM
Traum is offline   Trả Lời Với Trích Dẫn
The Following 2 Users Say Thank You to Traum For This Useful Post:
nghiepdu-socap (01-02-2013), nqt (02-02-2013)
Old 01-02-2013, 04:41 PM   #10
kien10a1
+Thành Viên+
 
kien10a1's Avatar
 
Tham gia ngày: Feb 2011
Đến từ: Vĩnh Yên- Vĩnh Phúc
Bài gởi: 371
Thanks: 43
Thanked 263 Times in 153 Posts
Gửi tin nhắn qua Yahoo chát tới kien10a1
Bài 2
[RIGHT][I][B]Nguồn: MathScope.ORG[/B][/I][/RIGHT]
 
__________________
Quay về với nơi bắt đầu
kien10a1 is offline   Trả Lời Với Trích Dẫn
Old 01-02-2013, 05:34 PM   #11
nguyentatthu
Super Moderator
 
Tham gia ngày: Nov 2007
Đến từ: BH
Bài gởi: 212
Thanks: 135
Thanked 345 Times in 92 Posts
Em xin đóng góp bài
Bài 12.Cho các số thực $a,b,c$ thoả ${a^2} + {b^2} + {c^2} = ab + bc + ca + 1$. Chứng minh rằng:
$${(a + b + c)^2} \le 4 + 3{\left( {ab + bc + ca} \right)^2} + 18abc$$
[RIGHT][I][B]Nguồn: MathScope.ORG[/B][/I][/RIGHT]
 
nguyentatthu is offline   Trả Lời Với Trích Dẫn
The Following 2 Users Say Thank You to nguyentatthu For This Useful Post:
Mr_Pi (03-02-2013), thaygiaocht (01-02-2013)
Old 01-02-2013, 08:57 PM   #12
talata1992
+Thành Viên+
 
Tham gia ngày: Jan 2013
Bài gởi: 8
Thanks: 1
Thanked 0 Times in 0 Posts
Em xin đóng góp bài này:
Trên mặt phẳng cho $3n$ điểm $A_1,A_2,\ldots,A_{3n}$ sao cho tam giác $A_1A_2A_3$ đều và với mọi $k, 1 \le k \le n-1$ các điểm $A_{3k+1}, A_{3k+2}, A_{3k+3}$ là trung điểm các cạnh của tam giác $A_{3k-2}A_{3k-1}A_{3k}$. Mỗi một điểm trong $3n$ điểm đã cho được tô bằng một trong hai màu cho trước. Chứng minh rằng nếu $n \ge 7$ thì có thể tìm được 4 điểm cùng màu là đỉnh của một tam giác cân. Điều khẳng định trên còn đúng không nếu $n=6$?
[RIGHT][I][B]Nguồn: MathScope.ORG[/B][/I][/RIGHT]
 

thay đổi nội dung bởi: novae, 01-02-2013 lúc 09:16 PM
talata1992 is offline   Trả Lời Với Trích Dẫn
Old 02-02-2013, 10:45 AM   #13
nqt
+Thành Viên+
 
Tham gia ngày: Mar 2010
Bài gởi: 30
Thanks: 61
Thanked 15 Times in 11 Posts
Trích:
Nguyên văn bởi Traum View Post

Bài 11: Tìm tất cả hàm số $f:Q^+\to Q^+ $ thỏa mãn: $f(2) = 2 $, $f(xy) = f(x)f(y) $ và $f(x+y)\le 4f(x) + 2013f(y) $ với mọi $x,y\in Q^+ $. (Độ khó = bài 2 IMO hay TST).
$Q^{+}$ có chứa 0 không hèn?
[RIGHT][I][B]Nguồn: MathScope.ORG[/B][/I][/RIGHT]
 
nqt is offline   Trả Lời Với Trích Dẫn
Old 02-02-2013, 11:11 AM   #14
mariakagawa
+Thành Viên+
 
Tham gia ngày: Feb 2013
Bài gởi: 7
Thanks: 3
Thanked 3 Times in 2 Posts
Trích:
Nguyên văn bởi kien10a1 View Post
Bài 2
Hướng của bạn đúng rồi nhưng chưa chính xác một tí vì $v_2 \left((2^n+a)^{2^n}-1 \right)=n+v_2(2^n+a-1) $
Nó không chia hết cho $2^{n+2}$ nếu $a=4k+3$. Nên không thể có $(2^n+a)^{2^n+a}\equiv (2^n+a)^{a}(mod2^{n+2})$ với mọi $a$ lẻ.
[RIGHT][I][B]Nguồn: MathScope.ORG[/B][/I][/RIGHT]
 
mariakagawa is offline   Trả Lời Với Trích Dẫn
Old 02-02-2013, 11:37 AM   #15
kien10a1
+Thành Viên+
 
kien10a1's Avatar
 
Tham gia ngày: Feb 2011
Đến từ: Vĩnh Yên- Vĩnh Phúc
Bài gởi: 371
Thanks: 43
Thanked 263 Times in 153 Posts
Gửi tin nhắn qua Yahoo chát tới kien10a1
Trích:
Nguyên văn bởi mariakagawa View Post
Hướng của bạn đúng rồi nhưng chưa chính xác một tí vì $v_2 \left((2^n+a)^{2^n}-1 \right)=n+v_2(2^n+a-1) $
Nó không chia hết cho $2^{n+2}$ nếu $a=4k+3$. Nên không thể có $(2^n+a)^{2^n+a}\equiv (2^n+a)^{a}(mod2^{n+2})$ với mọi $a$ lẻ.
Mình nghĩ là $v_2 \left((2^n+a)^{2^n}-1 \right)=n+v_2(2^n+a-1)+v_2(2^n+a+1)-1\geqslant n+2 $ chứ nhỉ, đây là công thức LTE dành riêng cho $p=2 $ mà, cái bạn áp dụng là của $p $ lẻ .

Bài 10
Ta giả sử rằng trong dãy đã cho có 39 số chính phương là $b_{i_1}>b_{i_2}>...>b_{i_{39}} $
Từ $1 $ đến $2017 $ có $44 $ số chính phương là $1^2,2^2,...,44^2 $.
Trong dãy này có $43 $ cặp số dạng $(a^2,(a+1)^2) $.
Xét việc bỏ $1 $ số ra khỏi dãy, ta thấy mất đi tối đa $2 $ cặp dạng $(a^2,(a+1)^2) $. Như vậy, nếu để có $39 $ số, ta bỏ ra $5 $ số thì vẫn còn ít nhất $43-10=33 $ cặp số dạng $(a^2,(a+1)^2) $ trong dãy.
Chú ý rằng hiệu hai số này là $2a+1 $ luôn lẻ.
Suy ra có ít nhất $33 $ cặp $(b_{i_l},b_{i_j}) $ là hai số chính phương liên tiếp với $j>l $ .
$b_{i_l}-b_{i_j}=a_{i_{l+1}}+...+ a_{i_j}>0 $ và là số lẻ.
Từ đó suy ra trong các số $a_{i_{l+1}},.., a_{i_j} $ có ít nhất một số lẻ.
Mỗi cặp trong 33 cặp trên cho ta một số $a_t $ nào đó lẻ, các $a_t $ này phân biệt, suy ra dãy $1,2,3,..,63 $ có ít nhất $33 $ số lẻ, điều này vô lí vì trong dãy chỉ có $32 $ số lẻ.
Suy ra đpcm.
[RIGHT][I][B]Nguồn: MathScope.ORG[/B][/I][/RIGHT]
 
__________________
Quay về với nơi bắt đầu

thay đổi nội dung bởi: kien10a1, 02-02-2013 lúc 11:40 AM
kien10a1 is offline   Trả Lời Với Trích Dẫn
The Following User Says Thank You to kien10a1 For This Useful Post:
nguoi_vn1 (03-02-2013)
Trả lời Gởi Ðề Tài Mới

Bookmarks


Quuyền Hạn Của Bạn
You may not post new threads
You may not post replies
You may not post attachments
You may not edit your posts

BB code is Mở
Smilies đang Mở
[IMG] đang Mở
HTML đang Tắt

Chuyển đến


Múi giờ GMT. Hiện tại là 04:19 AM.


Powered by: vBulletin Copyright ©2000-2024, Jelsoft Enterprises Ltd.
Inactive Reminders By mathscope.org
[page compression: 123.25 k/139.90 k (11.90%)]